{ "cells": [ { "cell_type": "markdown", "metadata": {}, "source": [ "# NAM 2019 pMuTT Workshop\n", "\n", "Instructions and materials for the \"Theory, Applications, and Tools for Kinetic Modeling\" workshop can be found on [our documentation page](https://vlachosgroup.github.io/pMuTT/nam_2019.html).\n" ] }, { "cell_type": "markdown", "metadata": {}, "source": [ "# Table of Contents\n", "\n", "| **1\\. [Virtual Kinetic Laboratory Ecosystem](#section_1)**\n", "\n", "| **2\\. [Useful Links](#section_2)**\n", "\n", "| **3\\. [Constants](#section_3)**\n", "\n", "|-- **3.1. [Access common constants in appropriate units](#section_3_1)**\n", "\n", "|-- **3.2. [Convert between units](#section_3_2)**\n", "\n", "|-- **3.3. [Convert between equivalent quantities](#section_3_3)**\n", "\n", "| **4\\. [Exercise 1](#section_4)**\n", "\n", "| **5\\. [Creating statistical mechanical objects using StatMech](#section_5)**\n", "\n", "|-- **5.1. [Supported StatMech models](#section_5_1)**\n", "\n", "|--|-- **5.1.1 [Translations](#section_5_1_1)**\n", "\n", "|--|-- **5.1.2. [Vibrations](#section_5_1_2)**\n", "\n", "|--|-- **5.1.3. [Rotations](#section_5_1_3)**\n", "\n", "|--|-- **5.1.4. [Electronic](#section_5_1_4)**\n", "\n", "|--|-- **5.1.5. [Miscellaneous](#section_5_1_5)**\n", "\n", "|-- **5.2. [Initializing StatMech modes individually](#section_5_2)**\n", "\n", "|-- **5.3. [Initializing StatMech modes using presets](#section_5_3)**\n", "\n", "| **6\\. [Plot Thermodynamic Quantities](#section_6)**\n", "\n", "| **7\\. [Exercise 2](#section_7)**\n", "\n", "| **8\\. [Creating empirical objects](#section_8)**\n", "\n", "|-- **8.1. [Inputting a NASA polynomial directly](#section_8_1)**\n", "\n", "|-- **8.2. [Fitting an empirical object to a StatMech object](#section_8_2)**\n", "\n", "| **9\\. [Input/Output](#section_9)**\n", "\n", "|-- **9.1. [Input via Excel](#section_9_1)**\n", "\n", "|-- **9.2. [Output via Thermdat](#section_9_2)**\n", "\n", "| **10\\. [Reactions](#section_10)**\n", "\n", "| **11\\. [Exercise 3](#section_11)**\n", "\n", "| **12\\. [Solutions](#section_12)**\n", "\n", "|-- **12.1. [Solution 1](#section_12_1)**\n", "\n", "|-- **12.2. [Solution 2](#section_12_2)**\n", "\n", "|-- **12.3. [Solution 3](#section_12_3)**" ] }, { "cell_type": "markdown", "metadata": {}, "source": [ "" ] }, { "cell_type": "markdown", "metadata": {}, "source": [ "# 1. Virtual Kinetic Laboratory Ecosystem\n", "\n", "\n", "\n", "\n", "\n", "\n", "- Estimates thermochemical and kinetic parameters using statistical mechanics, transition state theory\n", "- Writes input files for kinetic models and eases thermodynamic analysis\n", "- Implemented in Python\n", " - Easy to learn\n", " - Heavily used in scientific community\n", " - Object-oriented approach is a natural analogy to chemical phenomenon\n", "- Library approach allows users to define the starting point and end point\n", "\n", "\n" ] }, { "cell_type": "markdown", "metadata": {}, "source": [ "" ] }, { "cell_type": "markdown", "metadata": {}, "source": [ "# 2. Useful Links\n", "\n", "- [Documentation](https://vlachosgroup.github.io/pMuTT/): find the most updated documentation\n", "- [Issues](https://github.com/VlachosGroup/pmutt/issues): report bugs, request features, receive help\n", "- [Examples](https://vlachosgroup.github.io/pMuTT/examples.html): see examples" ] }, { "cell_type": "markdown", "metadata": {}, "source": [ "" ] }, { "cell_type": "markdown", "metadata": {}, "source": [ "# 3. Constants\n", "\n", "The [constants module](https://vlachosgroup.github.io/pMuTT/constants.html) has a wide variety of functions for constants and unit conversion." ] }, { "cell_type": "markdown", "metadata": {}, "source": [ "" ] }, { "cell_type": "markdown", "metadata": {}, "source": [ "## 3.1. Access common constants in appropriate units\n", "Below, we access Planck's constant in J s." ] }, { "cell_type": "code", "execution_count": 1, "metadata": {}, "outputs": [ { "name": "stdout", "output_type": "stream", "text": [ "h = 6.582119513926018e-16 eV s\n" ] } ], "source": [ "from pmutt import constants as c\n", "\n", "h1 = c.h('eV s', bar=True)\n", "print('h = {} eV s'.format(h1))" ] }, { "cell_type": "markdown", "metadata": {}, "source": [ "" ] }, { "cell_type": "markdown", "metadata": {}, "source": [ "## 3.2. Convert between units\n", "Below, we convert 12 atm of pressure to psi." ] }, { "cell_type": "code", "execution_count": 2, "metadata": {}, "outputs": [ { "name": "stdout", "output_type": "stream", "text": [ "12.0 atm = 176.35175185906093 psi\n" ] } ], "source": [ "from pmutt import constants as c\n", "\n", "P_atm = 12. # atm\n", "P_psi = c.convert_unit(num=P_atm, initial='atm', final='psi')\n", "\n", "print('{} atm = {} psi'.format(P_atm, P_psi))" ] }, { "cell_type": "markdown", "metadata": {}, "source": [ "" ] }, { "cell_type": "markdown", "metadata": {}, "source": [ "## 3.3. Convert between equivalent quantities\n", "Below, we convert 1000 wavenumbers (cm-1) to frequency." ] }, { "cell_type": "code", "execution_count": 3, "metadata": {}, "outputs": [ { "name": "stdout", "output_type": "stream", "text": [ "1000.0 cm-1 = 29979245800000.0 Hz\n" ] } ], "source": [ "from pmutt import constants as c\n", "\n", "wave_num = 1000. # cm-1\n", "freq = c.wavenumber_to_freq(wave_num) # Hz\n", "\n", "print('{} cm-1 = {} Hz'.format(wave_num, freq))" ] }, { "cell_type": "markdown", "metadata": {}, "source": [ "" ] }, { "cell_type": "markdown", "metadata": {}, "source": [ "# 4. Exercise 1\n", "\n", "Using `pmutt.constants`, calculate the dimensionless enthalpy (H/RT) using the following information:\n", "- H = 0.5 eV\n", "- T = 77 F" ] }, { "cell_type": "code", "execution_count": 4, "metadata": {}, "outputs": [], "source": [ "# Fill in your answer for Exercise 1 here\n" ] }, { "cell_type": "markdown", "metadata": {}, "source": [ "" ] }, { "cell_type": "markdown", "metadata": {}, "source": [ "# 5. Creating statistical mechanical objects using StatMech\n", "\n", "Molecules show translational, vibrational, rotational, electronic, and nuclear modes.\n", "\n", "" ] }, { "cell_type": "markdown", "metadata": {}, "source": [ "" ] }, { "cell_type": "markdown", "metadata": {}, "source": [ "## 5.1. Supported StatMech modes\n", "\n", "\n", "\n", "The StatMech object allows us to specify translational, vibrational, rotational, electronic and nuclear modes independently, which gives flexibility in what behavior you would like. Below are the available modes." ] }, { "cell_type": "markdown", "metadata": {}, "source": [ "" ] }, { "cell_type": "markdown", "metadata": {}, "source": [ "### 5.1.1. Translations\n", "- [``FreeTrans``](https://vlachosgroup.github.io/pMuTT/statmech.html#freetrans) - Translations assuming no intermolecular interactions" ] }, { "cell_type": "markdown", "metadata": {}, "source": [ "" ] }, { "cell_type": "markdown", "metadata": {}, "source": [ "### 5.1.2. Vibrations\n", "- [``HarmonicVib``](https://vlachosgroup.github.io/pMuTT/statmech.html#harmonicvib) - Harmonic vibrations\n", "- [``QRRHOVib``](https://vlachosgroup.github.io/pMuTT/statmech.html#harmonicvib) - Quasi rigid rotor harmonic oscillator. Low frequency modes are treated as rigid rotations.\n", "- [``EinsteinVib``](https://vlachosgroup.github.io/pMuTT/statmech.html#einsteinvib) - Each atom in the crystal vibrates as independent 3D harmonic oscillators\n", "- [``DebyeVib``](https://vlachosgroup.github.io/pMuTT/statmech.html#debyevib) - Improves upon ``EinsteinVib`` by considering simultaneous vibrations. Improves accuracy at lower temperatures." ] }, { "cell_type": "markdown", "metadata": {}, "source": [ "" ] }, { "cell_type": "markdown", "metadata": {}, "source": [ "### 5.1.3. Rotations\n", "- [``RigidRotor``](https://vlachosgroup.github.io/pMuTT/statmech.html#rigidrotor) - Molecule can be rotated with no change in bond properties" ] }, { "cell_type": "markdown", "metadata": {}, "source": [ "" ] }, { "cell_type": "markdown", "metadata": {}, "source": [ "### 5.1.4. Electronic\n", "- [``GroundStateElec``](https://vlachosgroup.github.io/pMuTT/statmech.html#groundstateelec) - Electronic ground state of the system\n", "- [``LSR``](https://vlachosgroup.github.io/pMuTT/statmech.html#linear-scaling-relationships-lsrs) - Linear Scaling Relationship to estimate binding energies using reference adsorbate" ] }, { "cell_type": "markdown", "metadata": {}, "source": [ "" ] }, { "cell_type": "markdown", "metadata": {}, "source": [ "### 5.1.5. Miscellaneous\n", "- [``EmptyMode``](https://vlachosgroup.github.io/pMuTT/statmech.html#empty-mode) - Default mode if not specified. Does not contribute to any properties\n", "- [``ConstantMode``](https://vlachosgroup.github.io/pMuTT/statmech.html#constant-mode) - Specify arbitrary values to thermodynamic quantities\n", "\n", "Using a ``StatMech`` mode gives you access to all the common thermodynamic properties.\n", "\n", "\n", "\n", "For this example, we will use a hydrogen molecule as an ideal gas:\n", "- translations with no interaction between molecules\n", "- harmonic vibrations\n", "- rigid rotor rotations\n", "- ground state electronic structure\n", "- no contribution from nuclear modes.\n", "\n", "" ] }, { "cell_type": "markdown", "metadata": {}, "source": [ "" ] }, { "cell_type": "markdown", "metadata": {}, "source": [ "## 5.2. Initializing StatMech modes individually" ] }, { "cell_type": "code", "execution_count": 5, "metadata": {}, "outputs": [ { "name": "stdout", "output_type": "stream", "text": [ "H_H2(T=298 K) = -618.6 kJ/mol\n", "S_H2(T=298 K) = 130.23 J/mol/K\n" ] } ], "source": [ "from ase.build import molecule\n", "from pmutt.statmech import StatMech, trans, vib, rot, elec\n", "\n", "H2_atoms = molecule('H2')\n", "\n", "'''Translational'''\n", "H2_trans = trans.FreeTrans(n_degrees=3, atoms=H2_atoms)\n", "\n", "'''Vibrational'''\n", "H2_vib = vib.HarmonicVib(vib_wavenumbers=[4342.]) # vib_wavenumbers in cm-1\n", "\n", "'''Rotational'''\n", "H2_rot = rot.RigidRotor(symmetrynumber=2, atoms=H2_atoms)\n", "\n", "'''Electronic'''\n", "H2_elec = elec.GroundStateElec(potentialenergy=-6.77,spin=0) # potentialenergy in eV\n", "\n", "'''StatMech Initialization'''\n", "H2_statmech = StatMech(name='H2',\n", " trans_model=H2_trans,\n", " vib_model=H2_vib,\n", " rot_model=H2_rot,\n", " elec_model=H2_elec)\n", "\n", "'''Calculate thermodynamic properties'''\n", "H_statmech = H2_statmech.get_H(T=298., units='kJ/mol')\n", "S_statmech = H2_statmech.get_S(T=298., units='J/mol/K')\n", "print('H_H2(T=298 K) = {:.1f} kJ/mol'.format(H_statmech))\n", "print('S_H2(T=298 K) = {:.2f} J/mol/K'.format(S_statmech))" ] }, { "cell_type": "markdown", "metadata": {}, "source": [ "" ] }, { "cell_type": "markdown", "metadata": {}, "source": [ "## 5.3. Initializing StatMech modes using presets\n", "\n", "Commonly used models can be accessed via [``presets``](https://vlachosgroup.github.io/pMuTT/statmech.html#presets). The currently supported models are:\n", "\n", "- [``idealgas``](https://vlachosgroup.github.io/pMuTT/statmech.html#ideal-gas-idealgas) - Ideal gases\n", "- [``harmonic``](https://vlachosgroup.github.io/pMuTT/statmech.html#harmonic-approximation-harmonic) - Typical for surface species\n", "- [``electronic``](https://vlachosgroup.github.io/pMuTT/statmech.html#electronic-electronic) - Only has electronic modes\n", "- [``placeholder``](https://vlachosgroup.github.io/pMuTT/statmech.html#placeholder-placeholder) - No contribution to any property\n", "- [``constant``](https://vlachosgroup.github.io/pMuTT/statmech.html#constant-constant) - Use arbitrary constants to thermodynamic properties\n" ] }, { "cell_type": "code", "execution_count": 6, "metadata": {}, "outputs": [ { "name": "stdout", "output_type": "stream", "text": [ "H_H2(T=298 K) = -618.6 kJ/mol\n", "S_H2(T=298 K) = 130.23 J/mol/K\n" ] } ], "source": [ "from ase.build import molecule\n", "from pmutt.statmech import StatMech, presets\n", "\n", "H2_statmech = StatMech(atoms=molecule('H2'),\n", " vib_wavenumbers=[4342.], # cm-1\n", " symmetrynumber=2,\n", " potentialenergy=-6.77, # eV\n", " spin=0.,\n", " **presets['idealgas'])\n", "\n", "'''Calculate thermodynamic properties'''\n", "H_statmech = H2_statmech.get_H(T=298., units='kJ/mol')\n", "S_statmech = H2_statmech.get_S(T=298., units='J/mol/K')\n", "print('H_H2(T=298 K) = {:.1f} kJ/mol'.format(H_statmech))\n", "print('S_H2(T=298 K) = {:.2f} J/mol/K'.format(S_statmech))" ] }, { "cell_type": "markdown", "metadata": {}, "source": [ "" ] }, { "cell_type": "markdown", "metadata": {}, "source": [ "# 6. Plot Thermodynamic Quantities\n", "Use [`pmutt.plot_1D`](https://vlachosgroup.github.io/pMuTT/visual.html#plot-1d) and [`pmutt.plot_2D`](https://vlachosgroup.github.io/pMuTT/visual.html#plot-2d) to plot any function with respect to 1 or 2 variables." ] }, { "cell_type": "code", "execution_count": 7, "metadata": {}, "outputs": [ { "data": { "image/png": "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\n", "text/plain": [ "
" ] }, "metadata": { "needs_background": "light" }, "output_type": "display_data" } ], "source": [ "import numpy as np\n", "from matplotlib import pyplot as plt\n", "from pmutt import plot_1D, plot_2D\n", "\n", "T = np.linspace(300., 500.)\n", "\n", "f1, ax1 = plot_1D(H2_statmech,\n", " x_name='T', x_values=T,\n", " methods=('get_H', 'get_S', 'get_G'),\n", " get_H_kwargs={'units': 'kcal/mol'},\n", " get_S_kwargs={'units': 'cal/mol/K'},\n", " get_G_kwargs={'units': 'kcal/mol'})\n", "f1.set_size_inches(6, 6)\n", "f1.set_dpi(200)\n", "plt.show()" ] }, { "cell_type": "markdown", "metadata": {}, "source": [ "" ] }, { "cell_type": "markdown", "metadata": {}, "source": [ "# 7. Exercise 2\n", "\n", "1. Create a ``StatMech`` object for ideal gas-phase H2O. The necessary inputs are given below.\n", "\n", "| Parameter | Value |\n", "|--------------------------------|------------------------------|\n", "| atoms | `molecule('H2O')` |\n", "| Potential Energy (eV) | -6.7598 |\n", "| Symmetry number | 2 |\n", "| Spin | 0 |\n", "| Vibrational Wavenumbers (cm-1) | 3825.434, 3710.264, 1582.432 |\n", "\n", "2. Calculate the Gibbs energy in eV for H2O at T = 500 K and P = 2 bar.\n", "\n", "3. Create a ``StatMech`` object for a Cu crystal using the ``DebyeVib`` model for the vibration mode and ``GroundStateElec`` model for electronic mode. The necessary inputs are given below.\n", "\n", "| Parameter | Value |\n", "|------------------------|------------|\n", "| Debye Temperature (K) | 310 |\n", "| Interaction energy (eV)| 0 |\n", "| Potential energy (eV) | -14.922356 |\n", "\n", "4. Plot the H (in eV) and S (in eV/K) for Cu between T = 300 - 700 K." ] }, { "cell_type": "code", "execution_count": 8, "metadata": {}, "outputs": [], "source": [ "# Fill in your answer for Exercise 2 here\n", "\n", "# 1.\n", "\n", "\n", "# 2.\n", "\n", "\n", "# 3.\n", "\n", "\n", "# 4.\n" ] }, { "cell_type": "markdown", "metadata": {}, "source": [ "" ] }, { "cell_type": "markdown", "metadata": {}, "source": [ "# 8. Creating empirical objects\n", "Currently, pMuTT supports [NASA polynomials](https://vlachosgroup.github.io/pMuTT/empirical.html#nasa) and [Shomate polynomials](https://vlachosgroup.github.io/pMuTT/empirical.html#shomate). They can be initialized in three ways:\n", "- passing in the polynomials directly\n", "- from a model (e.g. ``StatMech``, ``Shomate``) (``from_model``)\n", "- from heat capacity, enthalpy and entropy data (``from_data``)\n", "\n", "" ] }, { "cell_type": "markdown", "metadata": {}, "source": [ "" ] }, { "cell_type": "markdown", "metadata": {}, "source": [ "## 8.1. Inputting a NASA polynomial directly\n", "\n", "The H2 NASA polynomial from the [Burcat database](http://combustion.berkeley.edu/gri_mech/version30/files30/thermo30.dat) is represented as:\n", "\n", "```\n", "H2 TPIS78H 2 G 200.000 3500.000 1000.000 1\n", " 3.33727920E+00-4.94024731E-05 4.99456778E-07-1.79566394E-10 2.00255376E-14 2\n", "-9.50158922E+02-3.20502331E+00 2.34433112E+00 7.98052075E-03-1.94781510E-05 3\n", " 2.01572094E-08-7.37611761E-12-9.17935173E+02 6.83010238E-01 4\n", "```\n", "\n", "This can be translated to pMuTT syntax using:" ] }, { "cell_type": "code", "execution_count": 9, "metadata": {}, "outputs": [ { "name": "stdout", "output_type": "stream", "text": [ "H_H2(T=298 K) = -0.0010337769809016294 kcal/mol\n" ] }, { "data": { "image/png": "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\n", "text/plain": [ "
" ] }, "metadata": { "needs_background": "light" }, "output_type": "display_data" } ], "source": [ "from pmutt.empirical.nasa import Nasa\n", "\n", "# Initialize NASA polynomial\n", "H2_nasa = Nasa(name='H2',\n", " elements={'H': 2},\n", " phase='G',\n", " T_low=200., T_mid=1000., T_high=3500.,\n", " a_low=[2.34433112E+00, 7.98052075E-03, -1.94781510E-05,\n", " 2.01572094E-08, -7.37611761E-12, -9.17935173E+02,\n", " 6.83010238E-01],\n", " a_high=[3.33727920E+00, -4.94024731E-05, 4.99456778E-07,\n", " -1.79566394E-10, 2.00255376E-14, -9.50158922E+02,\n", " -3.20502331E+00])\n", "\n", "# Calculate thermodynamic quantities using the same syntax as StatMech\n", "H_H2 = H2_nasa.get_H(units='kcal/mol', T=298.)\n", "print('H_H2(T=298 K) = {} kcal/mol'.format(H_H2))\n", "\n", "# Show thermodynamic quantities vs. T\n", "T = np.linspace(200., 3500.)\n", "f2, ax2 = plot_1D(H2_nasa,\n", " x_name='T', x_values=T,\n", " methods=('get_H', 'get_S', 'get_G'),\n", " get_H_kwargs={'units': 'kcal/mol'},\n", " get_S_kwargs={'units': 'cal/mol/K'},\n", " get_G_kwargs={'units': 'kcal/mol'})\n", "f2.set_size_inches(6, 6)\n", "f2.set_dpi(200)\n", "plt.show()" ] }, { "cell_type": "markdown", "metadata": {}, "source": [ "" ] }, { "cell_type": "markdown", "metadata": {}, "source": [ "## 8.2. Fitting an empirical object to a StatMech object\n", "Empirical objects can be made directly using ``StatMech`` objects and the ``from_model`` method." ] }, { "cell_type": "code", "execution_count": 10, "metadata": {}, "outputs": [ { "data": { "image/png": "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\n", "text/plain": [ "
" ] }, "metadata": { "needs_background": "light" }, "output_type": "display_data" } ], "source": [ "H2_nasa = Nasa.from_model(name='H2',\n", " T_low=200.,\n", " T_high=3500.,\n", " model=H2_statmech)\n", "\n", "# Compare the statistical mechanical model to the empirical model\n", "f3, ax3 = H2_nasa.plot_statmech_and_empirical(Cp_units='J/mol/K',\n", " H_units='kJ/mol',\n", " S_units='J/mol/K',\n", " G_units='kJ/mol')\n", "f3.set_size_inches(6, 8)\n", "f3.set_dpi(200)\n", "plt.show()" ] }, { "cell_type": "markdown", "metadata": {}, "source": [ "" ] }, { "cell_type": "markdown", "metadata": {}, "source": [ "# 9. Input/Output\n", "pMuTT has more IO functionality than below. See this page for [supported IO functions](https://vlachosgroup.github.io/pMuTT/io.html)." ] }, { "cell_type": "markdown", "metadata": {}, "source": [ "" ] }, { "cell_type": "markdown", "metadata": {}, "source": [ "## 9.1. Input via Excel\n", "\n", "Encoding each object in Python can be tedious. You can read several species from Excel spreadsheets using [``pmutt.io.excel.read_excel``](https://vlachosgroup.github.io/pmutt/io.html?highlight=read_excel#pmutt.io.excel.read_excel). Note that this function returns a list of dictionaries. This output allows you to initialize whichever object you want using kwargs syntax. There are also [special rules that depend on the header name](https://vlachosgroup.github.io/pMuTT/io.html#special-rules).\n", "\n", "Below, we show an example importing species data from a spreadsheet and creating a series of NASA polynomials." ] }, { "cell_type": "code", "execution_count": 11, "metadata": {}, "outputs": [ { "name": "stdout", "output_type": "stream", "text": [ "[{'atoms': Atoms(symbols='N2', pbc=True, cell=[20.0, 20.0, 21.12998]),\n", " 'elec_model': ,\n", " 'elements': {'H': 0, 'N': 2},\n", " 'model': ,\n", " 'n_degrees': 3,\n", " 'name': 'N2',\n", " 'optional': 'atoms',\n", " 'phase': 'G',\n", " 'potentialenergy': -16.63,\n", " 'required': ('molecular_weight',\n", " 'vib_wavenumbers',\n", " 'potentialenergy',\n", " 'spin',\n", " 'geometry',\n", " 'rot_temperatures',\n", " 'symmetrynumber'),\n", " 'rot_model': ,\n", " 'symmetrynumber': 2.0,\n", " 'trans_model': ,\n", " 'vib_model': ,\n", " 'vib_wavenumbers': [2744.0]},\n", " {'atoms': Atoms(symbols='NH3', pbc=True, cell=[21.627662, 21.409596, 20.388297]),\n", " 'elec_model': ,\n", " 'elements': {'H': 3, 'N': 1},\n", " 'model': ,\n", " 'n_degrees': 3,\n", " 'name': 'NH3',\n", " 'optional': 'atoms',\n", " 'phase': 'G',\n", " 'potentialenergy': -19.54,\n", " 'required': ('molecular_weight',\n", " 'vib_wavenumbers',\n", " 'potentialenergy',\n", " 'spin',\n", " 'geometry',\n", " 'rot_temperatures',\n", " 'symmetrynumber'),\n", " 'rot_model': ,\n", " 'symmetrynumber': 3.0,\n", " 'trans_model': ,\n", " 'vib_model': ,\n", " 'vib_wavenumbers': [3565.26, 3565.26, 3433.9, 1727.23, 1727.23, 1132.51]},\n", " {'atoms': Atoms(symbols='H2', pbc=True, cell=[20.0, 20.0, 20.737166]),\n", " 'elec_model': ,\n", " 'elements': {'H': 2, 'N': 0},\n", " 'model': ,\n", " 'n_degrees': 3,\n", " 'name': 'H2',\n", " 'optional': 'atoms',\n", " 'phase': 'G',\n", " 'potentialenergy': -6.77,\n", " 'required': ('molecular_weight',\n", " 'vib_wavenumbers',\n", " 'potentialenergy',\n", " 'spin',\n", " 'geometry',\n", " 'rot_temperatures',\n", " 'symmetrynumber'),\n", " 'rot_model': ,\n", " 'symmetrynumber': 2.0,\n", " 'trans_model': ,\n", " 'vib_model': ,\n", " 'vib_wavenumbers': [4342.0]},\n", " {'elec_model': ,\n", " 'elements': {'H': 0, 'N': 2},\n", " 'model': ,\n", " 'name': 'N2(S)',\n", " 'phase': 'S',\n", " 'potentialenergy': -17.239999999999952,\n", " 'required': ('vib_wavenumbers', 'potentialenergy', 'spin'),\n", " 'vib_model': ,\n", " 'vib_wavenumbers': [2197.19, 360.415, 347.343, 335.674, 62.076, 32.1794]},\n", " {'elec_model': ,\n", " 'elements': {'H': 0, 'N': 1},\n", " 'model': ,\n", " 'name': 'N(S)',\n", " 'phase': 'S',\n", " 'potentialenergy': -9.339999999999975,\n", " 'required': ('vib_wavenumbers', 'potentialenergy', 'spin'),\n", " 'vib_model': ,\n", " 'vib_wavenumbers': [549.105, 538.56, 504.323, 475.805, 459.081, 410.018]},\n", " {'elec_model': ,\n", " 'elements': {'H': 1, 'N': 0},\n", " 'model': ,\n", " 'name': 'H(S)',\n", " 'phase': 'S',\n", " 'potentialenergy': -4.0,\n", " 'required': ('vib_wavenumbers', 'potentialenergy', 'spin'),\n", " 'vib_model': ,\n", " 'vib_wavenumbers': [1003.51, 625.55, 616.29]},\n", " {'elec_model': ,\n", " 'elements': {'H': 3, 'N': 1},\n", " 'model': ,\n", " 'name': 'NH3(S)',\n", " 'phase': 'S',\n", " 'potentialenergy': -20.42999999999995,\n", " 'required': ('vib_wavenumbers', 'potentialenergy', 'spin'),\n", " 'vib_model': ,\n", " 'vib_wavenumbers': [3491.08913,\n", " 3488.81949,\n", " 3364.52323,\n", " 1583.51571,\n", " 1582.0703,\n", " 1124.22477,\n", " 570.21231,\n", " 567.221471,\n", " 333.089624,\n", " 122.859345,\n", " 83.828555,\n", " 70.625115]},\n", " {'elec_model': ,\n", " 'elements': {'H': 2, 'N': 1},\n", " 'model': ,\n", " 'name': 'NH2(S)',\n", " 'phase': 'S',\n", " 'potentialenergy': -16.589999999999975,\n", " 'required': ('vib_wavenumbers', 'potentialenergy', 'spin'),\n", " 'vib_model': ,\n", " 'vib_wavenumbers': [3469.30034,\n", " 3381.05197,\n", " 1503.01629,\n", " 698.868804,\n", " 625.596094,\n", " 615.939783,\n", " 475.130224,\n", " 298.12001,\n", " 153.250139]},\n", " {'elec_model': ,\n", " 'elements': {'H': 1, 'N': 1},\n", " 'model': ,\n", " 'name': 'NH(S)',\n", " 'phase': 'S',\n", " 'potentialenergy': -13.20999999999998,\n", " 'required': ('vib_wavenumbers', 'potentialenergy', 'spin'),\n", " 'vib_model': ,\n", " 'vib_wavenumbers': [3403.12878,\n", " 718.177588,\n", " 710.58052,\n", " 528.525659,\n", " 415.195869,\n", " 410.130797]},\n", " {'elec_model': ,\n", " 'elements': {'H': 3, 'N': 1},\n", " 'model': ,\n", " 'name': 'TS1_NH3(S)',\n", " 'phase': 'S',\n", " 'potentialenergy': -19.23999999999995,\n", " 'required': ('vib_wavenumbers', 'potentialenergy', 'spin'),\n", " 'vib_model': ,\n", " 'vib_wavenumbers': [3453.41063,\n", " 3355.66992,\n", " 1723.84977,\n", " 1487.95484,\n", " 959.150723,\n", " 888.946198,\n", " 594.089439,\n", " 428.431136,\n", " 227.032386,\n", " 206.046727,\n", " 142.135856]},\n", " {'elec_model': ,\n", " 'elements': {'H': 2, 'N': 1},\n", " 'model': ,\n", " 'name': 'TS2_NH2(S)',\n", " 'phase': 'S',\n", " 'potentialenergy': -15.869999999999974,\n", " 'required': ('vib_wavenumbers', 'potentialenergy', 'spin'),\n", " 'vib_model': ,\n", " 'vib_wavenumbers': [3426.44484,\n", " 1293.72327,\n", " 922.830768,\n", " 660.966598,\n", " 525.595124,\n", " 496.837263,\n", " 330.674459,\n", " 290.278005]},\n", " {'elec_model': ,\n", " 'elements': {'H': 1, 'N': 1},\n", " 'model': ,\n", " 'name': 'TS3_NH(S)',\n", " 'phase': 'S',\n", " 'potentialenergy': -11.92999999999998,\n", " 'required': ('vib_wavenumbers', 'potentialenergy', 'spin'),\n", " 'vib_model': ,\n", " 'vib_wavenumbers': [1201.60487,\n", " 491.566416,\n", " 462.015502,\n", " 402.158904,\n", " 242.138726]},\n", " {'elec_model': ,\n", " 'elements': {'H': 0, 'N': 2},\n", " 'model': ,\n", " 'name': 'TS4_N2(S)',\n", " 'phase': 'S',\n", " 'potentialenergy': -14.669999999999952,\n", " 'required': ('vib_wavenumbers', 'potentialenergy', 'spin'),\n", " 'vib_model': ,\n", " 'vib_wavenumbers': [485.614, 392.977, 386.186, 280.943, 168.431]},\n", " {'elec_model': ,\n", " 'elements': {'H': 0, 'N': 0},\n", " 'model': ,\n", " 'name': 'RU(S)',\n", " 'nucl_model': ,\n", " 'phase': 'S',\n", " 'required': (),\n", " 'rot_model': ,\n", " 'trans_model': ,\n", " 'vib_model': },\n", " {'elec_model': ,\n", " 'elements': {'H': 0, 'N': 0},\n", " 'model': ,\n", " 'name': 'RU(B)',\n", " 'nucl_model': ,\n", " 'phase': 'S',\n", " 'required': (),\n", " 'rot_model': ,\n", " 'trans_model': ,\n", " 'vib_model': }]\n" ] } ], "source": [ "import os\n", "from pprint import pprint\n", "from pathlib import Path\n", "from pmutt.io.excel import read_excel\n", "from pmutt.empirical.nasa import Nasa\n", "\n", "# Find the location of Jupyter notebook\n", "# Note that normally Python scripts have a __file__ variable but Jupyter notebook doesn't.\n", "# Using pathlib can overcome this limiation\n", "try:\n", " notebook_folder = os.path.dirname(__file__)\n", "except NameError:\n", " notebook_folder = Path().resolve()\n", "os.chdir(notebook_folder)\n", "\n", "# Read the data from Excel\n", "ab_initio_data = read_excel(io='./input/NH3_Input_Data.xlsx', sheet_name='species')\n", "pprint(ab_initio_data)" ] }, { "cell_type": "code", "execution_count": 12, "metadata": {}, "outputs": [ { "name": "stdout", "output_type": "stream", "text": [ "Name Enthalpy (kcal/mol) Entropy (cal/mol/K) Gibbs energy (kcal/mol)\n", "--------------------------------------------------------------------------------\n", "N2 -377.5 45.9 -391.2\n", "NH3 -426.1 46.7 -440.0\n", "H2 -147.8 31.1 -157.1\n", "N2(S) -392.5 11.5 -396.0\n", "N(S) -224.0 -15.2 -219.5\n", "H(S) -90.4 -1.2 -90.0\n", "NH3(S) -445.8 13.4 -449.8\n", "NH2(S) -367.7 2.8 -368.5\n", "NH(S) -300.3 -4.3 -299.1\n", "TS1_NH3(S) -421.1 10.9 -424.4\n", "TS2_NH2(S) -355.6 1.9 -356.1\n", "TS3_NH(S) -274.4 -1.7 -274.0\n", "TS4_N2(S) -336.0 4.8 -337.4\n", "RU(S) 0.0 0.0 0.0\n", "RU(B) 0.0 0.0 0.0\n" ] } ], "source": [ "# Create NASA polynomials using **kwargs syntax\n", "nasa_species = []\n", "for species_data in ab_initio_data:\n", " single_nasa_species = Nasa.from_model(T_low=100.,\n", " T_high=1500.,\n", " **species_data)\n", " nasa_species.append(single_nasa_species)\n", "\n", "# Print out a table using enthalpy, entropy, Gibbs energy at 298 K for each species\n", "print('Name Enthalpy (kcal/mol) Entropy (cal/mol/K) Gibbs energy (kcal/mol)')\n", "print('--------------------------------------------------------------------------------')\n", "for single_nasa_species in nasa_species:\n", " name = single_nasa_species.name\n", " H = single_nasa_species.get_H(units='kcal/mol', T=298.)\n", " S = single_nasa_species.get_S(units='cal/mol/K', T=298.)\n", " G = single_nasa_species.get_G(units='kcal/mol', T=298.)\n", " print('{:12} {:10.1f} {:15.1f} {:15.1f}'.format(name, H, S, G))\n", " " ] }, { "cell_type": "markdown", "metadata": {}, "source": [ "" ] }, { "cell_type": "markdown", "metadata": {}, "source": [ "## 9.2. Output via Thermdat\n", "The thermdat format uses NASA polynomials to represent several species. It has a very particular format so doing it manually is error-prone. You can write a list of ``Nasa`` objects to thermdat format using [``pmutt.io.thermdat.write_thermdat``](https://vlachosgroup.github.io/pmutt/io.html#pmutt.io.thermdat.write_thermdat). \n", "\n", "Below, we write a thermdat file using the species imported from the spreadsheet." ] }, { "cell_type": "code", "execution_count": 13, "metadata": {}, "outputs": [], "source": [ "from pmutt.io.thermdat import write_thermdat\n", "\n", "write_thermdat(filename='./output/thermdat', nasa_species=nasa_species)" ] }, { "cell_type": "markdown", "metadata": {}, "source": [ "Similarly, a list of ``Nasa`` objects can be read from a thermdat using [``pmutt.io.thermdat.read_thermdat``](https://vlachosgroup.github.io/pMuTT/io.html#pmutt.io.thermdat.read_thermdat). " ] }, { "cell_type": "code", "execution_count": 14, "metadata": {}, "outputs": [], "source": [ "from pmutt.io.thermdat import read_thermdat\n", "\n", "nasa_species = read_thermdat('./output/thermdat')" ] }, { "cell_type": "markdown", "metadata": {}, "source": [ "" ] }, { "cell_type": "markdown", "metadata": {}, "source": [ "# 10. Reactions\n", "\n", "\n", "\n", "``Reaction`` objects can be created by putting together ``Nasa``, ``Shomate`` and ``StatMech`` objects.\n", "\n", "\n", "\n", "\n", "The ``from_string`` method is the easiest way to create a ``Reaction`` object. It requires the relevant species to be in a dictionary and a string to describe the reaction.\n", "\n", "\n", "\n", "We will demonstrate its use for the formation of NH3." ] }, { "cell_type": "code", "execution_count": 15, "metadata": {}, "outputs": [ { "name": "stdout", "output_type": "stream", "text": [ "Delta H_fwd(T = 300 K) = -16.1 kcal/mol\n", "Delta H_rev(T = 300 K) = 16.1 kcal/mol\n", "H_reactants(T = 300 K) = -410.5 kcal/mol\n" ] } ], "source": [ "from pmutt.empirical.nasa import Nasa\n", "from pmutt.empirical.shomate import Shomate\n", "from pmutt.reaction import Reaction\n", "\n", "# Create species. Note that you can mix different types of species\n", "species = {\n", " 'H2': StatMech(name='H2', atoms=molecule('H2'),\n", " vib_wavenumbers=[4342.], # cm-1\n", " symmetrynumber=2,\n", " potentialenergy=-6.77, # eV\n", " spin=0.,\n", " **presets['idealgas']),\n", " 'N2': Nasa(name='N2', T_low=300., T_mid=643., T_high=1000.,\n", " a_low=[3.3956319945669633, 0.001115707689025668,\n", " -4.301993779374381e-06, 6.8071424019295535e-09,\n", " -3.2903312791047058e-12, -191001.55648623788,\n", " 3.556111439828502],\n", " a_high=[4.050329990684662, -0.0029677854067980108,\n", " 5.323485005316287e-06, -3.3518122405333548e-09,\n", " 7.58446718337381e-13, -191086.2004520406,\n", " 0.6858235504924011]),\n", " 'NH3': Shomate(name='NH3', T_low=300., T_high=1000.,\n", " a=[18.792357134351683, 44.82725349479501,\n", " -10.05898449447048, 0.3711633831565547,\n", " 0.2969942466370908, -1791.225746924463,\n", " 203.9035662274934, 1784.714638346206]),\n", "}\n", "\n", "# Define the formation of water reaction\n", "rxn = Reaction.from_string('1.5H2 + 0.5N2 = NH3', species)\n", "\n", "# Calculate forward change in enthalpy\n", "H_rxn_fwd = rxn.get_delta_H(units='kcal/mol', T=300.)\n", "print('Delta H_fwd(T = 300 K) = {:.1f} kcal/mol'.format(H_rxn_fwd))\n", "\n", "# Calculate reverse change in enthalpy\n", "H_rxn_rev = rxn.get_delta_H(units='kcal/mol', T=300., rev=True)\n", "print('Delta H_rev(T = 300 K) = {:.1f} kcal/mol'.format(H_rxn_rev))\n", "\n", "# Calculate enthalpy of reactants\n", "H_react = rxn.get_H_state(units='kcal/mol', T=300., state='reactants')\n", "print('H_reactants(T = 300 K) = {:.1f} kcal/mol'.format(H_react))" ] }, { "cell_type": "markdown", "metadata": {}, "source": [ "" ] }, { "cell_type": "markdown", "metadata": {}, "source": [ "## 11. Exercise 3\n", "\n", "1. Use [``pmutt.io.thermdat.read_thermdat``](https://vlachosgroup.github.io/pMuTT/io.html#pmutt.io.thermdat.read_thermdat) to read the thermdat from './output/thermdat'.\n", "\n", "2. Convert the list of ``Nasa`` to a dictionary of ``Nasa`` using ``pmutt.pmutt_list_to_dict``. The syntax to use the function is shown below.\n", "\n", "```\n", "from pmutt import pmutt_list_to_dict\n", "\n", "species_dict = pmutt_list_to_dict(species_list)\n", "```\n", "\n", "3. Create a ``Reaction`` from the string: ``NH3(S) + RU(S) = TS1_NH3(S) = NH2(S) + H(S)`` and the dictionary from step 2.\n", "\n", "4. Calculate the forward reaction enthalpy in kcal/mol at 298 K using [``Reaction.get_delta_H``](https://vlachosgroup.github.io/pMuTT/reactions.html#pmutt.reaction.Reaction.get_E_act).\n", "\n", "5. Calculate the forward activation energy in kcal/mol at 298 K using [``Reaction.get_E_act``](https://vlachosgroup.github.io/pMuTT/reactions.html#pmutt.reaction.Reaction.get_E_act)." ] }, { "cell_type": "code", "execution_count": 16, "metadata": {}, "outputs": [], "source": [ "# Fill in your answer for Exercise 3\n", "\n", "# 1.\n", "\n", "\n", "# 2.\n", "\n", "\n", "# 3. \n", "\n", "\n", "# 4.\n", "\n", "\n", "# 5.\n", "\n" ] }, { "cell_type": "markdown", "metadata": {}, "source": [ "" ] }, { "cell_type": "markdown", "metadata": {}, "source": [ "# 12. Solutions" ] }, { "cell_type": "markdown", "metadata": {}, "source": [ "" ] }, { "cell_type": "markdown", "metadata": {}, "source": [ "## 12.1. Solution to Exercise 1\n", "[Link to Exercise 1](#section_4)" ] }, { "cell_type": "code", "execution_count": 17, "metadata": {}, "outputs": [ { "name": "stdout", "output_type": "stream", "text": [ "H/RT = 19.460878937648065\n" ] } ], "source": [ "from pmutt import constants as c\n", "\n", "# Define information given\n", "H = 0.5 # eV\n", "T = 77 # F\n", "\n", "# Calculate H/RT\n", "HoRT = H/c.R('eV/K')/c.convert_unit(77, initial='F', final='K')\n", "print('H/RT = {}'.format(HoRT))" ] }, { "cell_type": "markdown", "metadata": {}, "source": [ "" ] }, { "cell_type": "markdown", "metadata": {}, "source": [ "## 12.2. Solution to Exercise 2\n", "[Link to Exercise 2](#section_7)" ] }, { "cell_type": "code", "execution_count": 18, "metadata": {}, "outputs": [ { "name": "stdout", "output_type": "stream", "text": [ "G_H2O(T = 500 K, P = 2 bar) = -7.060978619777942 eV\n" ] }, { "data": { "image/png": "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\n", "text/plain": [ "
" ] }, "metadata": { "needs_background": "light" }, "output_type": "display_data" } ], "source": [ "from ase.build import molecule\n", "from pmutt import plot_1D\n", "from pmutt.statmech import StatMech, presets\n", "from pmutt.statmech.vib import DebyeVib\n", "from pmutt.statmech.elec import GroundStateElec\n", "\n", "# 1. Create H2O molecule\n", "H2O_statmech = StatMech(atoms=molecule('H2O'),\n", " potentialenergy=-6.7598,\n", " symmetrynumber=2,\n", " spin=0,\n", " vib_wavenumbers=[3825.434, 3710.264, 1582.432],\n", " **presets['idealgas'])\n", "\n", "\n", "# 2. Calculate Gibbs energy of H2O at T = 500 K and P = 2 bar\n", "T = 500. # K\n", "P = 2. # bar\n", "G_H2O = H2O_statmech.get_G(units='eV', T=T, P=P)\n", "print('G_H2O(T = 500 K, P = 2 bar) = {} eV'.format(G_H2O))\n", "\n", "\n", "# 3. Create Cu crystal\n", "Cu_vib = DebyeVib(debye_temperature=310., interaction_energy=0.)\n", "Cu_elec = GroundStateElec(potentialenergy=-14.922356)\n", "Cu_statmech = StatMech(vib_model=Cu_vib, elec_model=Cu_elec)\n", "\n", "\n", "# 4. Plot the 1D profile for H and S between 300 K and 700 K\n", "T = np.linspace(300., 700.) # K\n", "f2, ax2 = plot_1D(Cu_statmech,\n", " x_name='T', x_values=T,\n", " methods=('get_H', 'get_S'),\n", " get_H_kwargs={'units': 'eV'},\n", " get_S_kwargs={'units': 'eV/K'})\n", "f2.set_size_inches(6, 6)\n", "f2.set_dpi(200)\n", "plt.show()" ] }, { "cell_type": "markdown", "metadata": {}, "source": [ "" ] }, { "cell_type": "markdown", "metadata": {}, "source": [ "## 12.3. Solution to Exercise 3\n", "[Link to Exercise 3](#section_11)" ] }, { "cell_type": "code", "execution_count": 19, "metadata": {}, "outputs": [ { "name": "stdout", "output_type": "stream", "text": [ "H_rxn = -12.3 kcal/mol\n", "Ea_fwd = 24.6 kcal/mol\n" ] } ], "source": [ "from pmutt.io.thermdat import read_thermdat\n", "from pmutt import pmutt_list_to_dict\n", "from pmutt.reaction import Reaction\n", "\n", "# 1. Read the thermdat file\n", "species_list = read_thermdat('./output/thermdat')\n", "\n", "# 2. Convert the list of Nasa to a dictionary of Nasa\n", "species_dict = pmutt_list_to_dict(species_list)\n", "\n", "# 3. Create a Reaction from the specified string\n", "rxn = Reaction.from_string('NH3(S) + RU(S) = TS1_NH3(S) = NH2(S) + H(S)', species_dict)\n", "\n", "# 4. Calculate the reaction enthalpy\n", "H_rxn = rxn.get_delta_H(units='kcal/mol', T=298.)\n", "print('H_rxn = {:.1f} kcal/mol'.format(H_rxn))\n", "\n", "# 5. Calculate the forward activation energy\n", "Ea = rxn.get_E_act(units='kcal/mol', T=298.)\n", "print('Ea_fwd = {:.1f} kcal/mol'.format(Ea))" ] } ], "metadata": { "kernelspec": { "display_name": "Python 3", "language": "python", "name": "python3" }, "language_info": { "codemirror_mode": { "name": "ipython", "version": 3 }, "file_extension": ".py", "mimetype": "text/x-python", "name": "python", "nbconvert_exporter": "python", "pygments_lexer": "ipython3", "version": "3.7.6" } }, "nbformat": 4, "nbformat_minor": 2 }